Pre-Exam 2023 (part 4): erasable inks - our answers

The second claims part was about erasable inks.

“This invention provides erasable inks that provide good writing performance when used in writing instruments such as pens. The erasable inks are water-based inks that have a short "waiting time", i.e. inks that can be erased soon after they are applied on a surface made, for instance, of paper. Because the water-based inks are based on using water as a solvent rather than on using other solvents, the erasable inks of the invention are relatively non-toxic and odour-free.” [002] of the description of the client’s application)

Our answers to this second claims analysis part are given below. We have numbered the questions and statements for easy reference in the discussion.

During the exam, candidates had access to the EPO Legal Text pages, so including the Guidelines. Access was to the live versions, so to the versions in force on 17 March 2023 (so not the version of 31.10.2022 acc REE/IPEE).

Candidates could access the exam in all languages, English, French and German.

In the claims analysis parts (as in the legal parts), the order of the four statements of each of the questions was randomized, i.e., it was different for different candidates. 

In view of the randomized order, we provide the questions and statements together with our answers. If you check your answers with ours, note that the order of the questions and of the four statements from a single question may be different!!!

Please feel invited to comment!

The exam had a mix of common and less-common claims analysis topics. Part 4 was very very chemical with many function-defines agent and resins, many chemical names and formulas, ranges and an example in a table. It seemed that the paper lacked some definitions to also allow the non-chemist to understand it all in full, e.g., as to how much water is contained in a water-based solvent system, and whether an ester-based solvent system also includes water.

Please do not post your comments anonymously - it is allowed, but it makes responding more difficult and rather clumsy ("Dear Mr/Mrs/Ms Anonymous of 18-03-2022 18:03"), whereas using your real name or a nickname is more personal, more interesting and makes a more attractive conversation. You do not need to log in or make an account - it is OK to just put your (nick) name at the end of your post.

Note: the other claims part is discussed in another blog post: here; the legal part is discussed in another blog post: here; first impressions and general comments can be posted here.

Nico & Roel

 

Question 16

Claims as filed with the description on 16.08.2019

I.1. An erasable ink composition comprising:
(a) water;
(b) a colouring agent; and
(c) a releasing agent.

I.2. An erasable ink composition comprising:
(a) water;
(b) a colouring agent;
(c) a releasing agent; and
(d) a film-forming polymeric binder.

I.3. An erasable ink composition according to claim I.2, wherein the water is present in an amount ranging from about 40% to about 90% w/w.

I.4. An erasable ink composition according to claim I.2, wherein the colouring agent is present in an amount ranging from about 0.5% to about 30% w/w.

I.5. An erasable ink composition according to claim I.2, wherein the releasing agent includes a modified siloxane and is present in an amount ranging from about 0.3% to about 30% w/w.

I.6. An erasable ink composition according to claim I.2, wherein the polymeric binder is present in an amount ranging from about 1% to about 15% w/w and comprises a polyvinyl butyral resin.

I.7. An erasable ink composition according to claim I.2, wherein said erasable ink has an erasability of greater than 95% when erased within five minutes of applying the ink to the sheet of paper.

I.8. A pen comprising an outer body, a writing tip at one end of said body, a reservoir included within said body and connected to said writing tip, and within said reservoir a waterbased erasable ink composition comprising:
(a) water;
(b) a colouring agent in an amount of 1.5% to about 30% w/w;
(c) a releasing agent having a water solubility ranging from about 0.5% to about 70%
 of the erasable ink composition; and
(d) a film-forming polymeric binder.

For each of the statements, indicate whether the statement is true (T) or false (F):

16.1 The following feature is described as essential in the description: The modified siloxane-containing releasing agent comprises less than 1% w/w of colloidal silica.

The statement has no reference to a specific claim, so the statement cannot be unambiguously answered.

For embodiments or claims comprising modified siloxane, (for an even more improved erasing) it is essential; [006]: “When the releasing agent includes a modified siloxane, it has to contain less than 1% w/w of colloidal silica”.  This would result in T.

But modified siloxane is not essential (see 16.4) so that in other embodiments of without modified siloxane, it is also not essential that modified siloxane-containing releasing agent comprises comprises less than 1% w/w of colloidal silica. This would result in F.

Both T and F can this be argued.

16.2 The following feature is described as essential in the description: The polymeric binder is a polyvinyl butyral resin.

F – [008] “such as”

16.3 The following feature is described as essential in the description: The erasable ink composition comprises from about 1.5% to about 30% w/w of the colouring agent by weight of the erasable ink composition.

F – [005] “preferably”

16.4 The following feature is described as essential in the description: The releasing agent is a modified siloxane.

F – described as optional in [006]; only essential for an even more improved erasing [006], not for the effects in [005]

 

Question 17

After receipt of the search report from the EPO, the applicant envisages to replace claims I.1. and I.8. respectively as follow:

I.1. An erasable ink composition comprising:
(a) water;
(b) a colouring agent; and
(c) a releasing agent.

I.1bis. An erasable ink composition comprising:
(a) water;
(b) a colouring agent; and
(c) a film-forming polymeric binder.

I.8 . A pen comprising an outer body, a writing tip at one end of said body, a reservoir included within said body and connected to said writing tip, and within said reservoir a waterbased erasable ink composition comprising:
(a) water;
(b) a colouring agent in an amount of 1.5% to about 30% w/w;
(c) a releasing agent having a water solubility ranging from about 0.5% to about 70%
 of the erasable ink composition; and
(d) a film-forming polymeric binder.

I.8bis. A pen comprising an outer body, a writing tip at one end of said body, a reservoir included within said body and connected to said writing tip, and within said reservoir a waterbased erasable ink composition comprising:
(a) water in an amount ranging from about 40% to about 90% w/w;
(b) a colouring agent in an amount of 1.5% to about 30% w/w;
(c) a releasing agent having a water solubility ranging from about 0.5% to about 70%; and
(d) a film-forming polymeric binder.

For each of the statements, indicate whether the statement is true (T) or false (F):

17.1 Claim I.5 contains all the essential features.

F - For modified siloxane, it is essential that it contains less than 1% w/w of colloidal silica, which is not defined in claim I.5.

Note: it also contains non-essential features, such as the binder [004] via the dependence on I.2, and the features of claim I.,5 itself.

I.5, when written out, reads:
An erasable ink composition comprising:
(a) water;
(b) a colouring agent;
(c) a releasing agent; and
(d) a film-forming polymeric binder.
wherein the releasing agent includes a modified siloxane and is present in an amount ranging from about 0.3% to about 30% w/w.

17.2 Claim I.1bis, if filed as envisaged, would meet the requirement of Article 123(2) EPC.

F - It appears there needs to be a releasing agent ([004]), which is not included in claim I.1bis.

17.3 Claim I.4 meets the requirement of Article 84 EPC.

F - One aspect of Art. 84 is that all essential features are defined by the claim - according to [005], the minimum for the colouring agent is 0.75% w/w.
Another aspect is the use of 'about' which should be replaced by the numerical tolerances implied by 'about' – Guidelines.

I.4 when written out reads:
An erasable ink composition comprising:
(a) water;
(b) a colouring agent;
(c) a releasing agent; and
(d) a film-forming polymeric binder,
wherein the colouring agent is present in an amount ranging from about 0.5% to about 30% w/w.

17.4 Claim I.8bis, if filed as envisaged, would meet the requirement of Article 123(2) EPC.

F – Undisclosed combination of ranges.

 

Question 18

For each of the statements, indicate whether the statement is true (T) or false (F):

18.1 Claim I.1 is novel over D2.

F – Water is part of a 'water-based solvent system', colouring agent = 'pigment, whatever its color', releasing agent = 'dispersant ... have a releasing effect'

18.2 Claim I.1 is novel over D1.

F – Anticipated by D1's prior art section [002]: Water is part of 'water-based', colouring agent = 'pigment', releasing agent = 'releasing and spreading agents'

18.3 Claim I.3 is novel over D2.

T – D2 described “about 74% to about 94% w/w of the water-based solvent
system”, but it does not indicate which percentage of that is water and what percentage is pigment – could be 30% water and 64% pigment?

But facts seem to be missing: D2 has a film-forming dispersent but it is not clear that this is a 'polymeric binder'. The specific examples of polymeric binders do not match the specific examples of dispersants in D2.

Note possibly F is concluded by a chemist who, when using special knowledge, may (or not??) consider it implicit that a water-based solvent system cannot hold more than a few % of pigment – but that I nowhere disclosed and may not be true, or may at least not be necessarily so, which is needed for a novelty-destroying effect.

I.3 when written out reads:
 An erasable ink composition comprising:
(a) water;
(b) a colouring agent;
(c) a releasing agent; and
(d) a film-forming polymeric binder,
wherein the water is present in an amount ranging from about 40% to about 90% w/w.

18.4 Claim I.6 is novel over D2.

T – 2 certainly does not disclose polyvinyl butyral resin

I.6 when written out reads:
An erasable ink composition comprising:
(a) water;
(b) a colouring agent;
(c) a releasing agent; and
(d) a film-forming polymeric binder,
wherein the polymeric binder is present in an amount ranging from about 1% to about 15% w/w and comprises a polyvinyl butyral resin.

 

Question 19

For each of the statements, indicate whether the statement is true (T) or false (F):

19.1 D2 discloses a pen that contains an erasable ink composition.

T - See first sentence of [002]

19.2 D2 does not teach that the erasability of the erasable ink composition is dependent on the drying time.

T - Indeed, D2 lacks this teaching

19.3 D1 teaches that an essential element of all erasable ink compositions is the presence of a releasing agent.

F – [002] "Generally, such erasable ink compositions also include ... releasing and spreading agents" - "Generally" means it is not essential

19.4 D1 mentions that water in the solvent system has disadvantages.

F - It does not.


 

Question 20

Consider that the following new set of claims (II.1 to II.3) was filed by the applicant during the examination proceedings to expedite prosecution and the patent was granted five months ago with the following set of claims:

II.1. An erasable ink composition comprising:
(a) from about 40% to about 90% w/w of water;
(b) from about 1.5% to about 30% w/w of a colouring agent;
(c) from about 0.3% to about 30% w/w of a releasing agent; and preferably
(d) from about 1% to about 15% w/w of a film-forming polymeric binder.

II.2. An erasable ink composition comprising:
(a) from about 40% to about 90% w/w of water;
(b) from about 1.5% to about 30% w/w of a colouring agent;
(c) from about 0.3% to about 30% w/w of modified siloxane releasing agent.

II.3. An erasable ink composition, wherein said erasable ink has an erasability of greater than 95% when erased within five minutes of applying the ink to the sheet of paper.

For each of the statements, indicate whether the statement is true (T) or false (F):

20.1 In an opposition procedure, an opponent will likely be successful with a novelty attack against claim II.2 over D1.

F – D1 has two types of disclosures:
D1 [002] prior art water-based compositions: no modified siloxane releasing agent and no ranges;
D1 [003]-[004]: no water mentioned: ester-based solvent system, not water-based solvent.

20.2 In an opposition procedure, an opponent will likely be successful with an attack under Article 83 EPC against claim II.3.

F - The patent application discloses in [011] an erasable ink composition which would allow the skilled person to carry out the invention defined by claim II.3 since this composition reaches an erasability of 95% already after 10 seconds ([012])

Note: the claim itself is a 'result-to-be-achieved' claim and problematic under Art. 84 EPC but this is not a ground for opposition.

20.3 In an opposition procedure, an opponent will likely be successful with an attack under Article 123(2) EPC against claim II.2.

T - Not supported by the original claim set (e.g., 1.5% lower bound is missing in claim I.4). There is seemingly support in [009], but according to [006], when the releasing agent includes modified siloxane, it has to contain less than 1% w/w of colloidal silica. This essential feature is not in newly introduced claim II.2 -> intermediate generalization.

20.4 In an opposition procedure, an opponent will likely be successful with a novelty attack against claim II.1 over D2.

F - Facts seem missing: D2 has a film-forming dispersent but it is not clear that this is a 'polymeric binder'. The specific examples of polymeric binders do not match the specific examples of dispersants in D2. In addition, water-based solvent is present from about 74% to about 94% w/w, of which 74% falls within the claimed range of about 40% to about 90% w/w. However, it is not clear how much of the water-based solvent in D2 is actually water, so that D2 cannot  be a direct and unambiguous disclosure of some % w/w of water. Further, there is no direct and unambiguous disclosure of the same combination of the same ranges, nor of an individualized ink within the terms of the claim (similar as two-list selection) – hence not novelty-destroying [updated 20/3/2023]

Note: initially, we also had an argument on the binder, as we missed the "preferably" because it is at the end of feature (c) rather than at the start of feature (d), where one would expect it.

 We look forward to your comment! (Please use your name or a nickname)

(c) DeltaPatents


Annex – Pre-printables for Part 4 (copied from the pdf; added 19/3)

EUROPEAN QUALIFYING EXAMINATION 2023

Pre-examination

Documents for part 4
* Description of the invention
* Documents D1 and D2

Description of the invention
PCT 2021/321654, filed 16.08.2019

[001] Some writing instruments, e.g. pens and markers, include erasable inks which allow
markings formed with the ink to be erased efficiently. It is desirable that erasable inks
provide good writing performance when compared with non-erasable inks.

[002] This invention provides erasable inks that provide good writing performance when
used in writing instruments such as pens. The erasable inks are water-based inks that
have a short "waiting time", i.e. inks that can be erased soon after they are applied on a
surface made, for instance, of paper. Because the water-based inks are based on using
water as a solvent rather than on using other solvents, the erasable inks of the invention
are relatively non-toxic and odour-free.

[003] All the amounts are expressed, as per the convention in the technical area, as %
w/w, corresponding to a weight percent (%) of a given compound per weight of the
erasable ink composition.

[004] In one aspect, the invention features an erasable ink composition that includes
(a) water, (b) a colouring agent, (c) a releasing agent, and optionally (d) a film-forming
polymeric binder.

[005] The erasable ink compositions of the present invention preferably contain from about
1.0% to 30% w/w of the colouring agent, in the form of a pigment on a solid basis, more
preferably about 1.5% to 5% w/w of colouring agent. So to avoid that the colour intensity is
too weak for proper reading, the erasable ink has to contain more than 0.75% w/w of the
colouring agent.

[006] The releasing agent allows the erasable ink composition to be erased from a surface
made of paper. It is a finding of the inventors that if the releasing agent includes a modified
siloxane, the erasing is even more improved. When the releasing agent includes a
modified siloxane, it has to contain less than 1% w/w of colloidal silica. To enhance
erasability, it is necessary that the releasing agent has a water solubility ranging from
about 0.5% to 70%.

[007] The erasable ink compositions preferably contain from about 0.3% to 30% w/w of the
releasing agent, more preferably from about 1% to 7% w/w of said releasing agent. If the
amount of the releasing agent is too high, the erasable ink composition may smear,
whereas if the amount of the releasing agent is too low, then its erasability may be
compromised.

[008] When present, the polymeric binder is a resin, such as a polyvinyl-based resin (e.g. a
resin of PVC or of polyvinyl butyral (PVB)), an acrylic-based resin or a melamine resin.
The preferred polymeric binder includes a plasticised polyvinyl butyral resin. It allows the
erasable ink composition to shine under the light.

[009] Some implementations include one or more of the following features. The water is
present in an amount ranging from about 40% to about 90% w/w, the colouring agent is
present in an amount ranging from about 1.5% to about 30% w/w, the releasing agent is
present in an amount ranging from about 0.3% to about 30% w/w and the optional
polymeric binder is present in an amount ranging from about 1% to about 15% w/w,
wherein the percentages are by weight of the erasable ink composition.

[010] In another aspect, the invention relates to a pen comprising an outer body, a writing
tip at one end of said body, a reservoir included within said body and connected to said
writing tip, and within said reservoir a water-based erasable ink as described above.

[011] Example: An erasable ink composition was formed by mixing the ingredients shown
in Table 1, wherein the amounts are percentages by weight of the erasable ink
composition:

Ingredient

Amount (w/w)

Description

Water

60%

Solvent

Polyvinyl butyral

23%

Binder

Ethylhexyl Phthalate

8%

Plasticiser

Blue pigment

3.5%

Colouring agent

Polyethylene oxide modified
poly (dimethyl) siloxane

5%

Releasing agent

Oleic acid

0.5%

Emulsifier

[012] This ink was applied to a piece of paper. After 30 seconds, an eraser was used to
remove the ink of the paper. The ink erased cleanly, leaving no visible residue or staining.
A second marking was applied to the piece of paper and allowed to dry for a longer period
(60 seconds). This marking also erased cleanly. The ink exhibited an erasability of 95%
after a waiting time of 10 seconds, and an erasability of 100% when completely dry.

(No drawings with the application)

D1 – Advertisement in The Local Sun, published on the 15 August 2019

[001] For a number of years, there has been a desire to produce a pen that produces an
erasable, pencil-like trace. The particular choice of colouring pigments is important for
maintaining the erasability of traces formed by the erasable ink composition.

[002] Similarly, numerous water-based erasable ink compositions exist in the prior art.
Typically, such water-based compositions include a block copolymer such as styrenebutadiene and a pigment. Generally, such erasable ink compositions also include one or
more additives such as releasing and spreading agents, antioxidants, surfactants,
gelatinisers, lubricants and various waxes.

[003] The erasable ink composition of our new pen produces traces that are substantially
erasable with a common pencil eraser and are erasable for at least one day after the tracs
are formed on paper.

[004] Some specificities of our product are:
- The pigment, of any colour, is present in an amount of 2% to 25% weight per weight
(w/w).
- An organic ester-based solvent system is present in an amount of 25% to 55 % w/w.
- The solvent system comprises a component selected from the group consisting of an
optionally substituted cycloalkane, an optionally substituted cycloalkanone, an
optionally substituted cycloalkene, and mixtures thereof; and an organic ester. The
solvent system may comprise additionally other types of commonly known solvents.
- A polymeric binder being a liquid butene polymer combined to a plasticiser.

(No drawings with D1)

D2 – Editorial paper published in Pen Technology on 1 March 1995

[001] Erasable ink compositions, and more particularly erasable ink compositions for use
in writing instruments, are becoming more and more popular. However, erasable ink
compositions should not only be erasable but also provide good writing performance when
used in writing instruments.

[002] The simplest erasable ink composition for use in a writing instrument, such as a pen,
comprises a solvent and a pigment. Preferably, said pigment has a flake-like morphology.
The pigment, whatever its colour, is dispersed in the solvent. It seems that for the most
efficient systems, the erasable ink compositions include from about 5% to about 20% w/w
of the flake-like pigment and from about 74% to about 94% w/w of the water-based solvent
system. Such erasable ink compositions exhibit an erasability of at least 80%.

[003] Optionally, the erasable ink composition can comprise a dispersant. Typically, the
dispersants are water-soluble polymers that include polymeric chains. When present, the
dispersant is typically included at about 1% to 4% w/w, and most preferably about 1.5% to
2% w/w. Examples of suitable dispersants include, but are not limited to, nonionic
copolymers or anionic substituted alkoxylated polymers. One of the advantages of the
dispersant is that it forms a film at the surface of the writing as well as to have a releasing
effect.

(No drawings with D2)

Comments

  1. About the pen I was not sure : actually is discloses destined to be in a pan but not an actual pen comprising ink… so …

    ReplyDelete
    Replies
    1. Anyway thanks a lot

      Delete
    2. I think that the pen is not disclosed in D2 : the fact that the ink can be used in a pen doesn't imply that a pen filled with the ink has effectively been prepared, even for the measurement of erasability the conditions of which are note described.

      Delete
    3. It does: D2 [002] "The simplest erasable ink composition for use in a writing instrument, such as a pen, comprises a solvent and a pigment" discloses a pen using an erasable ink composition.

      Delete
    4. Does this mean if a prior art states e.g. "a wheel for use in a car", this anticipates a claim relating to a car?
      - E

      Delete
    5. I also applied the *strict* novelty analysis, and concluded D2 discloses an ink, not a pen.

      Delete
    6. I disagree with Roel, that D2 discloses a pen. As both Jorge, rbbc and Basile conclude that "for use in a writing instrument, such as a pen," doesn't disclose a pen per se with a container for an ink, etc.

      Delete
    7. I agree with Roel.

      If your reasoning would be correct, Ellie, the only way to disclose anything ever would be to use the exact same wording. Of course, "the use of a car" anticipates a car.

      Delete
  2. Not sure I agree that Q16.1 is ambiguous. The feature relates to the releasing agent when it contains siloxane, in which case it does need to contain less than 1% of colloidal silica. The question doesn't ask if the presence of a siloxane-containing releasing agent is necessary < if it did, on the same principle Q16.2 would be false solely for the reason that a polymeric binder is not essential. But Q16.2 clearly is asking if it's essential that [if present] the polymeric binder needs to be polyvinyl butyral resin, the same principle applies to Q16.1

    ReplyDelete
    Replies
    1. I think that the answer to Q16.1 should most likely be true because it is described as essential (for embodiments comprising modified siloxane) and as noted in the DP answer, there is no reference to a specific claim.

      Q16.2 is a different situation because (where present) the polymeric binder is a resin SUCH AS a polyvinyl butyral resin.

      Delete
    2. I also tend to think that the question is not ambiguous. For my understanding, it is not essential for the invention that the releasing agent contains siloxane. If that is the case, it must contain less than 1 % of colloidal silica; however, this is only necessary in the situation where the releasing agent contains siloxane, not for other types of releasing agents.

      Delete
  3. concerning answer 20.3 : the releasing agent doesn't '"include siloxane" but is "a modified siloxane agent", meaning 100% of siloxane. Therefore, it implicitely contains less than 1% w/w of colloidal silica. I think the answer is consequently false.

    ReplyDelete
    Replies
    1. Are you sure? F for 20.3 means that an oppo based on 123(2) will NOT be successful.

      Delete
    2. I confirm my first comment : there is no intermediate generalisation since there is no silice at all and only 100% of siloxane. Therefore an oppo won't be successful

      Delete
    3. I do not understand. “modified siloxane releasing agent” as an agent that releases modified siloxane, so there is modified siloxane in it. Isn’t ts?

      Delete
    4. Or maybe this is your point:

      “a releasing agent including a modified siloxane” (as in the description) is “a modified-siloxane releasing agent”,
      versus
      “a modified siloxane releasing agent” (without the dash, as in this claim) is “ a modified version of a releasing agent including siloxane” (I.e., no necessarily modified siloxane)?

      What a difference a dash can make!
      (Also see the case law on the effect of a missing comma…)

      Delete
    5. My point is perhaps related to the French translation : A modified siloxane agent is in french : un agent de siloxane which means 100 % siloxane. This is different from an agent including siloxane ( in French un agent comprenant du siloxane) which means up to 100 % siloxane.

      Delete
    6. I would second that argument. In the German version it reads "Trennmittel aus modifiziertem Siloxan". While not explicitily stated "aus" can only mean "bestehend aus" as in "consisting of" as the German equivalent of "comprising" does not make sense with the word "aus".

      Delete
    7. Hi Basti,

      I think you are wrong with this one. "aus" could also be "hergestellt aus", which I do not think is limitting. Also, I think this combination of ranges is only dislcosed in par [009], which also demands the polymeric binder, that is not present in the claim.

      Delete
    8. I also interpret "a modified siloxane releasing agent" as "a modified siloxane forming the releasing agent" or "a releasing agent which is modified siloxane".

      For example, "a metal stick" describes that metal forms a stick, i.e., the stick consists of metal. However, a stick could comprise wood and metal.

      Delete
  4. For Q. 20.4, isn't the polymeric binder disclosed in Claim II.1 optional because of the "preferably" language, and so whether the disclosure in D2 relates to a "polymeric binder" does not matter?

    ReplyDelete
    Replies
    1. I thought the same, feature (d) of II.1 claim is optional, so the question to whether II.1 is novel over D2 or not boils down to how much water is in the water-based solvent...

      In the exam I considered that water could also be considered a water-based solvent (no idea if that is really true?) and though that if that was the case it would most likely be obvious to a skilled person. But it is a good point that it might be not a clear and unambiguous disclosure.

      Delete
    2. Additional argument I considered in the exam why water-based solvent can be considered to be pure water was because in the filed application example table water was labelled as solvent

      Delete
    3. I thought the same also, but I agree with the contention that it is not clear how much of the water-based solvent in D2 is actually water, because a water-based solvent can also contain ingredients other than water, information must be given to calculate the water contained in the water-based solvent.
      One more issue, does not D2 teach that the dispersants have a releasing function, such that it can be considered as a releasing agent of claim II.1.
      And I would say the answer to question 20.4 is True.

      Delete
    4. What we considered relevant is (see our answer): "However, it is not clear how much of the water-based solvent in D2 is actually water, so that D2 cannot be a direct and unambiguous disclosure of some % w/w of water – hence not novelty-destroying."

      Delete
    5. About "water-based solvent system", I think there might be some direction in the description of the invention: "Because the water-based inks are based on using water as a solvent rather than on using other solvents, the erasable inks of the invention are relatively non-toxic and odour-free."

      From this, one could come to the conclusion that a water-based solvent system is consisting of water as a solvent. Especially non-chemists must have come to that conclusion, I think. As a chemist, I would personally think of solvent mixtures (like water + ethanol) when reading "water-based solvent system", but there is nowhere an indication in the direction of solvent mixtures, so I decided to go for the simple conclusion: water-based means water.

      In D2, it says:
      "It seems that for the most efficient systems, the erasable ink compositions include from about 5% to about 20% w/w of the flake-like pigment and from about 74% to about 94% w/w of the water-based solvent system."

      Combined with the statement that the simplest ink comprises a pigment and a solvent, I concluded that the ink in D2 contains 74-94% w/w of water.

      Delete
    6. I agree what a water-based solvent “comprises” water. But how much? The term “-based” directly implied that the solved comprises more than just water. Otherwise, it would have said “that uses water as a solvent”.

      Delete
    7. I went with true for 20.4 because the "preferably" meant that the polymeric binder was not limiting and entirely optional (GL-F-IV, 4.16), so the only question was whether a water based solvent system could be understood to mean water. Given the context of the description of the invention in which water is described as being the solvent, I understood water-based to mean water.

      Perhaps this question will be neutralized.

      Delete
    8. BG, I followed your reasoning entirely.

      According to colleagues working on inks: when "water-based" is used, the solvent is regarded as 100% water. If one wants to indicate that the majority of the solvent is water, one should use "aqueous" instead.
      But the different terminologies are still often up for debate.

      I stick to my previous comment that the description of the document as filed directs towards the conclusion that water-based means water:
      "water-based inks are based on using water as a solvent rather than on using other solvents".

      I think the answer is true.

      But they should definitely have avoided such terms in the exam, in my opinion.

      Delete
    9. At the EQE, I am sorry for your colleagues, but their special knowledge is not relevant in view of Rule 22(3) IPREE, last sentence:
      "Candidates shall not use any special knowledge they may have of the technical field of the invention."

      So, under Rule 22(3) IPREE, first sentence "Candidates shall accept the facts given in the examination paper and limit themselves to those facts" (which is also interpreted as "the paper shall have all the facts needed to answer it in a clear and unambiguous way" - see Case Law of the Disciplinary Board of Appeal), the paper should have indicated that/whether a "water-based solvent" is regarded as 100% water.

      Delete
    10. @ BG: I agree with you that "preferably" is entirely optional, and the reference to the binder in our answer could maybe better have bee omitted.

      However:

      I also agree with you that (in the absence of a definition to the contrary) the term “-based” directly implies that the water-based solvent comprises more than just water - this there is no direct and unambiguous disclosure of the water % in D2 - this alone is already enough to render the claim novel.

      Further, and most importantly, to be novelty destroying, there shall be a direct and unambiguous disclosure of a single example/embodiment in D2 that falls within the terms of the claim.
      But there is not such a disclosure, also not if we would assume that a water-based solvent is 100% water:

      D2 shows in [002]: an erasable ink composition, including
      - from about 5% to about 20% w/w of the flake-like pigment and
      - from about 74% to about 94% w/w of the water-based solvent system.
      This is clearly not novelty-destroying as it has no releasing agent.

      D2 shows in [003]: an erasable ink composition, including
      - from about 5% to about 20% w/w of the flake-like pigment and
      - from about 74% to about 94% w/w of the water-based solvent system and
      - dispersant (=releasing agent) is typically included at about 1% to 4% w/w, and most preferably about 1.5% to 2% w/w
      - with examples of some specific polymeric dispersants.
      However, although tempting to read multiple disclosures in this set of ranges, this does NOT disclose: an erasable ink composition including
      - 5% (or 20%) of the flake-like pigment
      - 74% of the water-based solvent system and
      - 1% (or 1.5, 2, 4%) w/w of polymeric dispersant (= releasing agent)
      because this would be a multiple selection from a plurality of ranges similar as a 3-list selection – see GL G-8, (i).
      In the absence of no individualized examples nor any examples with just a single range, D2 is not novelty-destroying.

      Delete
    11. Similarly can one not arbitrarily combine each of the following from D2 [003], as they all are optional/ typical (so not "necessarily", but "in most case" - unspecified which):

      "Typically, the dispersants are water-soluble polymers that include polymeric chains" -- so can be different.
      " the dispersant is typically included at about 1% to 4% w/w, and most preferably about 1.5% to 2% w/w" -- is "typical", but not necessarily these values for specific types of dispersant.
      "Examples of suitable dispersants include, but are not limited to, nonionic
      copolymers or anionic substituted alkoxylated polymers." -- note that a non-chemist cannot even conclude wheter these include polymeric chains are (also) different types of polymers; water-soluble seems implicit as it is suitable for an ink; for none of these two groups of polymers is a specific combination made with specific %-ranges.
      So, a specific combination like "a dispersant included at 1.5% w/w, the dispesant being an anionic substituted alkoxylated polymer" is NOT disclosed - that would be analogous to a two-list selection or maybe even a non-disclosed combination of features.

      Delete
    12. @Fabienne:
      I donot understand why a “water-based solvent” would be 100% water (and how we can know that), especially as a “water-based ink” ([002]) clearly is not 100% water… Does the term “water-based” have two different meanings in the same document and while both relating to solutions or solution-like ink?
      If we can derive something from this comparison, then it is that “water-based solvent” necessarily has more than just water…

      Delete
    13. I see your point.

      My understanding from the description of the application as filed was that when an ink is water-based, it uses water as a solvent rather than any other solvents ([002]). It is true that there need to be other ingredients comprised in the ink, as the word "ink" is already implicitly disclosing colorant (otherwise it would be varnish or something similar, non-colored).
      Extrapolating the "definition" from water-based in the description as filed ([002]) to D2 ([002]), a water-based solvent system would be a solvent system using water as a solvent, rather than any other solvents.

      Just a thought: what if it would have been "pigment-based ink, which is an ink based on using pigments rather than other colorants" vs. a "pigment-based colorant system". Would you conclude that the pigment-based colorant system comprises something else than pigment? Personally, I wouldn't. IF however, the description would have said: "based on using MOSTLY pigment", or in the case of the paper "MOSTLY water", then I would not conclude that water-based means 100% water.


      PS. As a chemist, I read through the paper in about 10 minutes and I found it relatively easy to understand (the questions were still giving me head aches though). My first thought was that this was incredibly unfair for non-chemists. But as you also indicate above, there are some risks related to that, since I would use knowledge from my field to come to conclusions that aren't necessarily meant the way they are (cfr. also the word "binder", which is a word that is used - often incorrectly perhaps - as a general broad term). They should just stick to sponges and yoga mats and stay away from specific terminology.

      Delete
    14. @Roel:

      From one of your previous posts, I don't understand this part:

      "However, although tempting to read multiple disclosures in this set of ranges, this does NOT disclose: an erasable ink composition including
      - 5% (or 20%) of the flake-like pigment
      - 74% of the water-based solvent system and
      - 1% (or 1.5, 2, 4%) w/w of polymeric dispersant (= releasing agent)
      because this would be a multiple selection from a plurality of ranges similar as a 3-list selection – see GL G-8, (i).
      In the absence of no individualized examples nor any examples with just a single range, D2 is not novelty-destroying."

      The question is not about these individual values, but about overlapping ranges? Isn't the embodiment of [003] still a single embodiment, defined in terms of ranges? According to my reasoning, all these ranges are novelty-destroying ranges for the ranges of claim II.1.

      If we assume that we can strike through part (d) because of the "preferably", then we end up with the following claim:
      II.1. An erasable ink composition comprising:
      (a) from about 40% to about 90% w/w of water;
      (b) from about 1.5% to about 30% w/w of a colouring agent;
      (c) from about 0.3% to about 30% w/w of a releasing agent;


      Then I concluded that claim II.1 was anticipated by [003] of D2,
      because:
      - about 74% to about 94% w/w of the water-based solvent system takes away the novelty of (a), because not-novel range, assuming water-based would mean water of course.
      - about 5% to about 20% w/w of the flake-like pigment takes away the novelty of (b), because not-novel range.
      - dispersant (=releasing agent) included at about 1% to 4% w/w takes away novelty from (c), same reason

      I found that the ranges were not novel because the person skilled in the art would seriously contemplate working in those ranges. (and they are not far removed from, nor narrow compared to etc.)

      Thanks in advance!

      Delete
    15. Or is only a single range allowed in one embodiment? (and why?)

      Delete
    16. Also, a polymer always contains polymeric chains. There is no such thing as a polymer not containing a polymeric chain. I have no idea why they included this bit of info. It's like saying "Typically, dogs are mammals that have mammary glands".

      But does it really matter I wonder?

      D2 says that a dispersant is optionally used, and that a dispersant has the advantage of having a releasing effect (= releasing agent).
      So, does it even matter what the dispersant looks like for the novelty of claim II.1, if it has a releasing effect?

      So the simplification I made in my head looks as follows:

      A composition comprising:
      - A in an amount from 74-95 w/w%
      - B in an amount from 5-20 w/w%
      - optionally also C in an amount from typically 1-4 w/w%, more preferably 1,5-2 w/w%.

      is, I think, novelty destroying for:

      A composition comprising:
      - A in an amount from 40-90 w/w%
      - B in an amount from 0,3-30 w/w%
      - C in an amount from 1,5-30 w/w%


      My apologies if I am obsessing too much over this :-D and thanks again for your answers!

      Delete
    17. Ho Fabienne,

      Maybe I shall rephrase my argument.
      My argument is that a prior art range discloses just its end points (and examples) and novelty of a claimed range is destroyed by an explicitly mentioned end-point of the known range, explicitly mentioned intermediate values or a specific example of the prior art in the overlap - see GL G-VI, 8 (iii) (in the 2023 edition; the section has been somewhat reshuffled and rewritten compared to the 2023 edition).

      With that in mind, the disclosure of D2 in [003] of an erasable ink composition, including
      - from about 5% to about 20% w/w of the flake-like pigment and
      - from about 74% to about 94% w/w of the water-based solvent system and
      - dispersant (=releasing agent) is typically included at about 1% to 4% w/w, and most preferably about 1.5% to 2% w/w
      is actually a disclosure of an erasable ink composition, including
      - the flake-like pigment at a w/w selected from a first list of 5% and 20% (2 entries) and
      - the water-based solvent system at a w/w selected from a second list of 74% and 94% w/w (2 entries) and
      - dispersant (=releasing agent) is included at a w/w selected from a third list of 1% , 4%, 1.5% and 2% (4 entries).
      Any specific combination from the list elements would be a two-list selection, and hence a non-disclosed combination.
      As there is not a single individual disclosure in D2, it cannot be novelty destroying for the claim.

      Delete
    18. I think T 653/93, cited in Case Law Book I.C.6.3.3 "Selection of parameter ranges - Multiple selection", is similar:

      "In case T 653/93 the appellant argued that the process of claim 1 was novel as it referred to a combination of three process features with selected ranges and product features with specific limits, a combination not disclosed in the prior art document. The board of appeal emphasised that in such situations the question of novelty could not be answered by contemplating the ranges of the various parameters separately. This would, in the board's judgment, be an artificial and unjustified approach, since it was not the specified ranges of the three parameters or their agglomeration that formed the subject-matter of claim 1, but the group of processes defined by the combination of these ranges, which was rather small when compared with the group of processes disclosed in the prior art document. Thus the claimed group of processes, characterised by the combination of three specific process parameters, was not explicitly disclosed in the prior art document and therefore could be said to result from a "multiple (i.e. threefold) selection". [...] It followed that the subject-matter of claim 1 was not considered as having been disclosed in the prior art document (see also T 1095/18)."

      To cite from the decision:
      I. Independent Claim 1 reads:
      "Process for the production of process oils with an aromatic content of more than 50 weight % according to ASTM D 2007 and a content of polycyclic aromatic compounds of less than 3 weight % according to IP 346 characterised in that primary extract, obtained by treatment of a lubricating oil distillate originating from a mineral oil, is extracted in a counter current extraction column with a polar solvent wherein the volume ratio of the primary extract feed to the polar solvent is in the range 1:1 to 1:1.8 and the head temperature in the counter current extraction column is 50-90 C and the bottom temperature in the column is 20-60 C and wherein the head temperature is higher than the bottom temperature."

      reason 3.1. Document (1) discloses a multi-step process for the treatment of a mineral-lubricating oil in which a primary extract resulting from a first extraction step undergoes in a second step a counter current extraction with furfural in an extraction column. While designated as being "preferred" the following numerical ranges are the only ones actually disclosed in document (1) for the respective process parameters:
      - a CHT of from 40 C to 100 C,
      - a column bottom temperature (CBT) of from 35 C to 80. C, and
      - a VR of 100 : 50-250, i.e. 1 : 0.5-2.5.
      3.2.1. First of all, it has to be emphasised that in situations like the present one the question of novelty cannot be answered by contemplating the ranges of the various parameters separately. This would be, in the Board's judgement, an artificial and unjustified approach, since not the specified ranges of the three respective parameters or their agglomeration form the subject-matter of Claim 1, but the group of processes defined by the combination of these ranges, which is rather small when compared with the group of processes disclosed in document (1).

      3.2.2. Bearing in mind this principle, it is to be noted that the presently claimed group of processes, which are characterised by the combination of the three process parameters
      - a CHT of 50 to 90 C,
      - a CBT of 20 to 60 C, and
      - a VR of 1:1 to 1:1.8,
      was not explicitly disclosed in document (1). This citation contains no pointer to the particular combination of the ranges given for the three parameters which fall completely (CHT and VR) or in part (CBT) within the respective ranges explicitly disclosed in document (1). The group of processes of Claim 1 of the application in suit can also be said to result from a "multiple (i.e. threefold) selection". [...]
      3.3. It follows that the subject-matter of Claim 1 was not disclosed in document (1).

      Delete
    19. Whether this is Pre-Exam level may be doubted.... especially as it needed some digging in the case law book to find explicit confirmation... Would even have been a challenge in Paper B or C...

      Delete
    20. Dear Roel, thank you so much for your answers and your effort!

      I am wondering whether the committee really expected us to come to this conclusion...

      Moreover, in T 653/93, the reason that there was an appeal in the first place, was that the examining division found claim 1 not to be novel!! So Fabienne's (and my) answer would have been in line with the examining division in this case, which I think is not too bad for a candidate after two years of training... :-)

      Also, is it reasonable to justify an answer in such an exam with a T-decision only? I am sure that somewhere, there is another T-decision, coming to the opposite conclusion?!

      Delete
    21. Thank you Roel for the detailed answers and looking that up!

      I checked T653/93 and saw that there was an unpredictable technical effect which was also taken into account, which is not the case with our erasable ink compositions.
      Moreover, they explicitly mention the following in T653/93 : "it was not the specified ranges of the three parameters or their agglomeration that formed the subject-matter of claim 1, but the group of processes defined by the combination of these ranges, which was rather small when compared with the group of processes disclosed in the prior art document."
      One could hardly say that the claimed ranges in Claim II.1 are giving rise to a small group of individual compositions compared to the group of compositions disclosed in the prior art. On the contrary even.

      Also the Guidelines do mention this:
      "As with the selection of a sub-range, it is not sufficient to exclude specific novelty-destroying values known from the prior-art range, it must also be considered whether the skilled person, in the light of the technical facts and taking into account the general knowledge in the field, would seriously contemplate applying the technical teaching of the prior-art document in the range of overlap."

      and then for example for overlapping chemical formulae (which I think can be also applied here perhaps, although with a bit of a stretch), the GL say:

      "Novelty is acknowledged if the claimed subject-matter is distinguished from the prior art in the range of overlap by a new technical teaching, see T 12/90, point 2.6 of the Reasons. There is a new technical teaching if certain technical elements are new in comparison to the prior-art disclosure."

      I am really curious to see what the official answer and reasoning will be (and also if they regard "water-based" as "based on water rather than any other solvent").

      Delete
    22. Hi LJ and Fabienne,
      I did not cite the T-decision as "THE" legal basis for my answer, but as an "objective" example of my reasoning, as the Guidelines put it only in general terms in G-VI, 8: "In determining the novelty of a selection, it has to be decided whether the selected elements are disclosed in an individualised (concrete) form in the prior art".
      G-VI, 8(i)(c) also gives: " Examples of such [novel)] selections from two or more lists are the selection of: [...]
      (c) sub-ranges of several parameters from corresponding known ranges."
      Note that, in G-VI, 8, only (i)(c) relates to combinations of a plurality ranges (as in the question: 3) - tests for novelty of an individual sub-range of an indivisual prior art range are thus not relevant.

      I am also really curious to see what the official answer and reasoning will be and whether the statement maybe will be neutralized.

      NB: clearly a"water-based solvent" is "a solvent based on water rather than any other solvent" in the sense of "a solvent comprising water", but the question is whether it was intended to mean "a solvent being water" -- as discussed, there is no basis for the latter interpretation in the paper (rather to the contrary in view of "water-based ink" not being an "ink being water") and an answer shall thus not need to rely in it, while it seems from your inquiry at your chemical colleagues that chemists would consider it to be the latter (own knowledge, Rule 22(3) IPREE). Also here, curious to see whether all statements relying on "water-based solvent" being equal or not equal to "water" will be neutralized.

      Delete
    23. As far as the water-based reasoning goes...

      Even if "water-based solvent system" could mean "solvent comprising water", at least HALF of the solvent should be water, as otherwise the majority of the solvent system would be solvent resulting in it to be "solvent-based".

      This would imply that "from about 74% to about 94% w/w of the water-based solvent system" discloses at least "from about 37% to about 47% water", in which case it is still an overlapping range (and not narrow compared to the disclosed one).

      I still think "water-based" etc. is already debatable in a specialist context, let alone in a pre-exam context.

      (and this discussion is also relevant for question 18.3 btw)

      Delete
    24. Could it be that some of the inspiration for part 4 came from EP3239254B1?

      In par [0012], this document says: "The term "water-based" in this context means that in the case of the ink according to the invention in the solvent system of water and organic solvents, the water has a (substantially) greater proportion than the organic solvents." If that is the case, this would clearly confirm Fabiennes reasoning, that "74% to about 94% w/w of the water-based solvent system" discloses at least "from about 37% to about 47% water", which would then be novelty destroying.

      I totally understand: Special knowledge should not be considered. But does this mean that a question where any specialist in a corresponding field would give answer A, would have to be answered with an obviously wrong answer B, simply because some commonly known fact is not provided? Where does this stop, where does it begin?

      If the claim says 4, and the prior art says 2+2, would the answer be that the claim is novel, because knowing that 2+2=4 is special knowledge for a mathematician?

      What I find interesting about EP3239254B1 is that it provides definitions for all relevant components, such as pigments, binder, solvent...

      Delete
  5. Regarding the question 17.4, the caselaw T 2/81 2 states that to read "The disclosure of a quantitative range of values (e.g. for concentrations or temperatures) together with an included preferred narrower range also directly discloses the two possible part-ranges lying within the overall range on either side of the narrower range. Hence a simple combination of the preferred narrower range and one of these part-ranges is also unequivocally derivable and is supported by the disclosure."

    According to this caselaw, the preferred range from 1.5 to other point would generate two other part-ranges which is 0.3% to 1.5% and "other point" to 30%.

    Further, according to the caselaw T 2/81 r.3 to read "The simple sub-combination of these part-ranges of the concentration values as claimed would not merit novelty as “selection”, so that the restriction does not represent any new subject‑matter within the meaning of Article 123(2)."
    Therefore, the combination of the part-range and the preferred range would be possibly combined. Therefore, the range 1.5 to 30 % would not be considered as a new subject-matter within the meaning of Art. 123(2) EPC.

    Isn't it correct?

    ReplyDelete
    Replies
    1. I agree and also answered True for this question for the reasons you mentioned. Furthermore, [009] also discloses "one or more" of the following features.

      Delete
    2. I would also like to highlight that para. [009] says "ONE OR MORE" of the following features, which in my view means that the feature "40 to 90 % water" can be individually extracted and combined with claim I.8 to arrive at claim I.8bis without violating 123(2)

      Delete
    3. The above post was by me, I forgot to put my name

      Delete
    4. I agree with the above comments; surely 17.4 (Claim I.8bis, if filed as envisaged, would meet the requirement of Article 123(2) EPC) should be true.

      Delete
    5. The added water range was only disclosed with a vague indication to “in some embodiments”, but there is no direct and unambiguous disclosure of the combination of the subject-matter of original claim I.8 with this water-range. The amendment is thus a non-disclosed combination.

      Delete
    6. However, I respectfully disagree because it did not say as in some embodiments" but "some implementations include one or more of the following features" which clearly indicates that this is not the specific embodiment which is limited to the followed features but the implementation of the embodiments with the followed features. Therefore, the skilled person would that it is directly and unambiguous to just bring "one" feature of water weight range from the suggested embodiment (this might be considered as a selection from the list when it is a selection from the single list? Which would be considered as disclosed as well). and combine with the obviously disclosed features. This would not cause the issue of reservoir just because we picked one feature from the suggested implementation since it was clearly suggested from the description that "some implementations include one or more following features".

      Delete
    7. I can understand your argument as far as you mean “I.8 is independent claim so the term ‘in embodiments relate to embodiments of that”, but…

      Do you consider “one or more of the
      following F1, F2, F3 and F4” to disclosure all of 4! combinations F1, F1+F2, F1+F3, F1+F4, F1+F2+F3, F1+F2+F4, F1+F3+Fz4, F1+F2+F3+F4, F2, F2+F3, F2+F3, … disclosed?
      If so, why are this directly and unambiguously disclosed (when also remembering e.g. the two-lost principle for disclosure)?
      If not, why would then I.8+F1 be allowed?

      Delete
    8. Considering the further combinations more than one, I think it would be considered as equivalent to a twofold selection from the two identical list as stated in the following case law (T 811/96 r.1.6) to say "A selection of two components from one list is in fact equivalent to a twofold selection from two identical lists" which is considered to be new. Therefore the combination of more than two features would not be considered as directly and unambiguously disclosed from the said implementation example but choosing one feature would be derivable in my opinion.
      Therefore, by following the Guidelines G-VI 8(i) "If the prior art discloses elements in an individualized (concrete) form (a list), then a selection from a single list of specifically disclosed elements does not confer novelty (see T 12/81). However, if a selection from two or more lists of a certain length has to be made in order to arrive at a specific combination of features then the resulting combination of features, not specifically disclosed in the prior art, confers novelty (the "two lists principle").", it should be considered as a selection from a single list of specifically disclosed implementation.
      In short, I think even though the description stated that "one or more" features would be possibly adopted into the embodiment, only one choosing one feature to adopt to the embodiment would be allowed. Otherwise, it would be considered as an added-subject matter under the "two lists principle".

      Delete
    9. I agree, If it says „one or more of the following features“ followed by a list of 4 features, than at least those 4 features individually are directly disclosed. We do not need a combination of those features to arrive at the amended claim, so the two list principle does not apply here in my opinion

      Delete
    10. I also agree. I selected true for this reason.

      Delete
  6. Thank you very much for your answers.

    I answered 17.4 for T for following reasons:
    First of all, I understood the question 17 so that I.8bis replaces I.8 and I.8 is filed with the application. Comparing I.8bis with I.8, i) the range of water (about 40% to about 90% w/w) was added and ii) the term "of the erasable ink composition" was deleted.

    As to the amendment i): this amendment can be based on [009] as [009] says "some implementations include one or more of the following feature". For me, this sentence allows an implementation further to have one of the following feature. claim I.8 is an implementation and this implementation can have one of the following feature, i.e. the range of water (about 40% to about 90% w/w). As "including one of the fallowing features" is allowed, the water range does not have to be combined with other features disclosed in [009].

    As to the amendment ii) the deletion of "of the erasable ink composition" is redundant.

    I may make a mistake, so I will be thankful, if you can share your thoughts.

    ReplyDelete
    Replies
    1. I had exactly the same line of thinking.

      Delete
    2. Yes, I would also say it is true.

      Delete
    3. I disagree:

      Guidelines in H-V, 3.2.1:

      When evaluating whether the limitation of a claim by a feature extracted from a combination of features fulfils the requirements of Art. 123(2), the content of the application as filed must not be considered to be a reservoir from which individual features pertaining to separate embodiments can be combined in order to artificially create a particular combination.

      Delete
    4. See my comment above: we consider the term “in some embodiments” not sufficient to directly and unambiguously disclose the combination given in the amended claim.
      Comments are welcome!

      Delete
    5. I also thought exactly this way. I don't see this as a reservoir-type situation.

      Delete
  7. Thank you very much! Imo, The water-based solvent should have been described anywhere. All in all, I found the claim analysis part more difficult than it was in previous years.

    ReplyDelete
  8. Thanks a lot for sharing the answers! I really appreciated it!

    ReplyDelete
  9. The claim part was way harder than the previous years … way way harder the questions depended on your way of thinking you could not just say True or False… without discussing

    ReplyDelete
    Replies
    1. I totally agree… way too hard for the PRE examination imho. I think passed but it is still frustrating.

      Delete
    2. I fully agree. Not all all comparable to 2022, 2021 and 2019. Orders of magnitude more difficult. Part 4 level way beyond wo year experience.

      Delete
    3. I agree that part 4 is by far the most difficult claims analysis part since we have two claims analysis parts. The level at which “directly and unambiguously disclosed” (for Art.123(2)/amendments and for novelty/ state of the art) is very high - and amendments are already difficult in a True/False format (the committee once said “thou shall not think too deep”).

      Also, the art.100(b)/83 opposition in the claims part is surprising as Art.100(b) is not to be considered in
      Paper C / then why in a lower-experience True/False Pre-Exam paper (where the answers also relies on considering a single example can be -and in this case is- sufficient for the skilled person for sufficiency)

      Delete
    4. Harder than any of the previous exams, so frustrating. Not at all a pre-exam level claim analysis.

      Delete
  10. Thanks for you analysis! For Q20.2 I took 'within 5 minutes' to cover the range of 1-10 seconds, which isn't disclosed as having 95% erasibility - my reasoning was then that there was support across the full range of the claim and therefore a sufficiency attack may be successful?

    I also fully agree that this section seemed much harder than previous years. I also think the heavy focus on chemistry unfairly disadvantaged candidates who do not have this background. I spent a lot of time trying to decipher the meaning of the chemical words and groups, and figure out what they included or excluded. At the very least this wasted a lot of time and led to general confusion and stress.

    Thanks,
    P

    ReplyDelete
    Replies
    1. *wasn't support across the whole range

      P

      Delete
    2. I also had the same thought process. Within 5 minutes includes 1-10 seconds, which based on the disclosure, would not be 95% or greater.

      Delete
    3. I also wondered what a 95% erasibility means (maybe write 20 letters and one remains after erasing? :D). Hence it is not disclosed sufficiently clear and complete to be carried out by a person skilled in the art.
      Thanks, SE

      Delete
    4. For 20.2, I thought it was not clear since "the sheet" didn't have anticedent basis. Is this not not enough for a ground of opposition? E

      Delete
    5. For sufficiency, one should not just read the claim (as one would do for clarify), but also the description and drawings and also the skilled person’s common general knowledge:

      GL F-III, 1. Sufficiency of disclosure:

      “A detailed description of at least ONE way of carrying out the invention must be given. Since the application is addressed to the person skilled in the art, it is neither necessary nor desirable that details of well-known ancillary features are given, but the description must disclose any feature essential for carrying out the invention in sufficient detail to render it apparent to the skilled person how to put the invention into practice […] There are some instances where even a very broad field is sufficiently exemplified by a limited number of examples or even one example (see also F‑IV, 6.3). In these latter cases the application must contain, in addition to the examples, sufficient information to allow the person skilled in the art, using common general knowledge, to perform the invention over the whole area claimed without undue burden and without needing inventive skill (see T 727/95). In this context, the "whole area claimed" is to be understood as substantially any embodiment falling within the ambit of a claim, even though a limited amount of trial and error may be permissible, e.g. in an unexplored field or when there are many technical difficulties (see T 226/85and T 409/91).”

      Case Law typically uses the following wording:

      “ Article 83 EPC requires that the invention be disclosed in a manner sufficiently clear and complete for it to be carried out by a person skilled in the art. In view of this article, it is not problematic that claims - as is typically the case - represent a generalisation of the teaching of the description and the drawings.

      Decision G 1/03, point 2.5.2 further establishes that a claim might encompass non-working embodiments and still be allowable as long as the specification "contains sufficient information on the relevant criteria for finding appropriate alternatives over the claimed range with a reasonable effort". Point 2.5.2 of G 1/03 refers to the case law represented by, for instance, T 301/87, according to which "it is not necessary for the purpose of Articles 83 and 100(b) EPC that the disclosure of a patent is adequate to enable the skilled man to carry out all conceivable ways of operating the invention which are embraced by the claims" (point 3.2). T 515/00, cited by the proprietor, comes to the same conclusion.”
      (See e.g. T 1794/16 () of 20.10.2022 and T 0163/16 () of 20.10.2022, to mention just a few recent decisions).

      The Boards also emphasize that:

      “ As variously stated in case law, the disclosure is aimed at the person skilled in the art who may rely on common general knowledge to supplement the information contained in the patent. Textbooks and general technical literature form part of the common general knowledge, see Case Law of the Boards of Appeal, 10th edition (CLBA), II.C.4.1.” (E.g., in T 0804/21 () of 22.12.2022 and T 0428/21 (Chewing gum compositions comprising cannabinoids / SANAMMAD) of 9.2.2023 to just mentions a few recent decisions).

      In our case, the patent gives a clear example in the last paragraph as well as information on the relevant criteria for finding appropriate alternatives over the claimed range with a reasonable effort, when using -wheee necessary- also common general knowledge on erasable inks.

      Delete
    6. Part 4 was super ambiguous for multiple questions, but this one had me going back and forth between T/F the most, my reasoning in the end is based on the section of F-III 1. you left out:

      "A single example may suffice, but where the claims cover a broad field, the application is not usually regarded as satisfying the requirements of Art. 83 unless the description gives a number of examples or describes alternative embodiments or variations extending over the area protected by the claims. However, regard must be had to the facts and evidence of the particular case. There are some instances where even a very broad field is sufficiently exemplified by a limited number of examples or even one example (see also F‑IV, 6.3). In these latter cases the application must contain, in addition to the examples, sufficient information to allow the person skilled in the art, using common general knowledge, to perform the invention over the whole area claimed without undue burden and without needing inventive skill (see T 727/95)."

      The only section referring to erasability and drying times was 011/012 and only a single example was given. So according to the section of the GL above, the first requirement of supplying multiple embodiments or alternatives for a broad claim (I considered this claim very broad) is not met. Then T 727/95 requires sufficient information to carry out the invention over the whole claimed scope. I cannot find such information in the application as, again, erasability and drying time is only referred to in 011/012. The common knowledge (contained in the application, D1, and D2) does not give information about other inks concerning erasability and drying time and how one could experiment with them. I would argue that the skilled person is left alone with finding out what parameters actually influence drying time and erasability.

      Based on this, I put TRUE (i.e., a A.83 based oppo would be succesful)

      As a personal note, a

      Delete
    7. oops, last sentence got swallowed up: as a personal note, if I submitted a claim like this to the EPO, A. 83 would only be one of the grounds they would beat me with this left and right

      Delete
    8. "If I submitted a claim like this to the EPO, A. 83 would only be one of the grounds they would beat me with this left and right"

      Why would you think so? There is a very clear example that is enabling for the claim. Further erasability is a well-known term, so the skilled perspn knows how to measure that. An "erasable ink has an erasability of greater than 95% when erased within five minutes of applying the ink to the sheet of paper" cannot be misunderstood: the skilled person applies the ink to the sheet of paper, then waits for 5 minutes, and then measures the erasability - no undue burden, very clear instructions, this enabled.

      NB: If you would consider erasability to not be defined objectively, that would be an Art.84 issue, not an Art. 83 aspect, and you shall also not misuse a potential Art.84 issue to make an Art.83 attack.

      Delete
    9. The last line was rather out of frustration about the question. What I meant to say above is that erasability and drying times are disclosed for the exemplary ink composition of 011. Now the whole claimed scope of II.3 is supposedly larger than just the one example. But how does one find other ink compositions that have an erasability of 95% after 10 Minutes? Is this maybe the only composition with such properties and the scope of the claim is thus much smaller (I suppose not but how can we know from the description)? In my opinion the skilled person is not given any instructions in how to proceed to find other ink combinations that fall into the scope of the claim. T 727/95 clearly says that "in these latter cases [i.e., broad scope and only a single embodiment] the application must contain, in addition to the examples, sufficient information to allow the person skilled in the art, using common general knowledge, to perform the invention over the whole area claimed without undue burden and without needing inventive skill". Where is this information, i.e., where are instructions to perform the invention over the whole claimed scope? I couldn't find them. I'm not a chemist so maybe such experimenting is common in the art but that should have been disclosed somewhere.

      Delete
    10. Reading your comment again, I think I missed your point the first time that the process from 012 is enabling over the whole claimed scope. However, don't you think that the skilled person would have to apply inventive skill in the experiments? What I mean is, what parameters in the ink actually control erasability and drying time? What ingredients in the composition do I need to change to arrive at an ink composition that falls within the scope?

      Delete
    11. Exactly what I was thinking!!

      Delete
    12. Basti, I totally agree with your view and this is also why I chose „True“ for this question in the exam. Another aspect on this: It is also not described how „erasability“ may be determined. So if the skilled person wished to produce an erasable ink which has an erasability of 95% after 5 seconds (that is, within 5 minutes), the skilled person does not learn from the description how to determine the percentage of erasability ans thus, if such erasability is achieved by using the disclosed example.

      Delete
    13. Hi A,

      "It is also not described how „erasability“ may be determined." is an Art. 84 issue, and not Art.83.

      Further, the prior art also uses the concept "erasability" so that concept is known - even though the exact values are not well-defined, but again that is an Art. 84 issue.

      GL F-III, 11 "Sufficiency of disclosure and clarity":
      "In particular (see T 593/09), where a claim contains an ill-defined ("unclear", "ambiguous") parameter (see also F-IV, 4.11) and where, as a consequence, a person skilled in the art would not know whether they were working within or outside of the scope of the claim, this, by itself, is not a reason to deny sufficiency of disclosure as required by Art. 83. Nor is such
      a lack of clear definition necessarily a matter for objection under Art. 84
      only. What is decisive for establishing insufficiency within the meaning of
      Art. 83 is whether the parameter, in the specific case, is so ill-defined that a
      person skilled in the art is not able, on the basis of the disclosure as a
      whole and using common general knowledge, to identify (without undue
      burden) the technical measures necessary to solve the problem underlying
      the application at issue, e.g. see T 61/14.
      There is a delicate balance between Art. 83 and Art. 84, which has to be
      assessed on the merits of each individual case. Care has therefore to be
      taken in opposition that an insufficiency objection is not merely a hidden
      objection under Art. 84, especially in the case of ambiguities in the claims
      (T 608/07)."

      Note that the above is maybe one of the reasons why Art. 83/Art.100(b) is not used as a ground for opposition in Paper C and I was much surprised that the Pre-Exam claims part now did have it...

      In my opinion, the detailed example of [011]&[012] is a clearly enabled example, which also clearly solved the problem of providing an "erasable ink has an erasability of greater than 95% when erased within five minutes of applying the ink to the sheet of paper", as " ink exhibited an erasability of 95% after a waiting time of 10 seconds".
      As a result, there is one enabling disclosure within the claim scope -- a somewhat unclear meaning of erasability has no effect in view of the GL-citations above.
      As erasability is a known concept (see e.g., D2 [002]), the skilled person will be able to check erasability of every erasable ink that he can make - so without undue burden, the skilled person know whether he acts within the scope of the claim (apart from the Art.84 unclarity that makes the boundary somewhat vague).

      Note herein also GL F-III, 11: "Normally, therefore, an ambiguity in a claim will lead to an objection under Art. 83 ONLY IF the whole scope of the claim is affected, in the sense that it is impossible to carry out at all the invention defined therein. Otherwise an objection under Art. 84 is appropriate (see T 608/07, T 1811/13)."
      Clearly, NOT the whole scope of affected, as the example of [011]-[012] is in its scope (and even far from the boundaries): so at least the part of the claim that corresponds to the example is not affected. So the Art.84 aspect does not render the claim not-enabled.

      Delete
    14. I think it is somewhat likely that it’s impossible to achieve 95% after 1 s, as this is not supported by the application.
      Therefore, even if the claim would be hypothetically unambiguous, the skilled person would not be able to carry out the invention within 1 s.

      Further, the claim is unclear because it doesn’t specify how to broadly carry out (essential features) the invention to achieve an effect. The effect itself is clear and described as ranges (<5 min and >95%).
      Therefore, the ambiguity of the claim doesn’t change the scope (of the ranges) that needs to be carried out/achieved by the skilled person and GL F-III, 11 doesn’t apply here. F-III, 11 is for preventing that an ambiguity (what ink composition) increases the scope to be carried out, i.e., “a person skilled in the art would not know whether they were working within or outside of the scope of the claim”.

      Delete
    15. Question is not 1 second, but 5 minutes.

      Unclear is not Article 83.

      Delete
    16. Very good point, LE!

      The claim says when erased WITHIN five minutes, so erasing after one second is encompassed by the wording of the claim. As erasing 95% after one second clearly is not possible, the claim cannot be carried out in the entire range. Thus, an A 83 attack would be successful.

      Even erasing immediately after writing would also be encompassed by the claim.

      Delete
    17. The claim just says erased WITHIN five minutes. It does not require erasing
      within 1 second as a necessary feature.

      Delete
    18. Of course erasing within 1 second is not required, you are absolutely right. But erasing within 1 second is a possibility that is covered by the scope of the claim, among all other possibilites, e.g., erasing after four minutes.

      According to A83, it must be possible to carry out the claim over its entire scope, which imo is not possible when erasing after 1 second.

      Delete
    19. I think this is exactly where Roel's argument differs from ours. If I understood correctly, Roel is saying that the single embodiment is sufficient because starting from this single embodiment and experimenting with the ingredients is not undue burden for the skilled person. If this was the case, then a single example for the broad scope would constitute a sufficient disclosure. My argument would be that no instructions are given on how to get to an ink that is 95% erasable instantly, after 1 second, after 2 seconds, ... (all covered by the scope) and that it is not even clear if such inks exist. In my eyes it is undue burden for the skilled person to find such inks as the description does not give any hints whatsoever.

      Delete
    20. Exactly what I am thinking, Basti.

      In fact, during the exam I erroneously read within 5 seconds, instead of 5 minutes, and immediately concluded that there is no sufficient disclosure. After the first shock that I made a stupid mistake, I soon came to the conclusion that my reasoning was not too bad, actually. I really think that there is no disclosure for such short times, at least it's highly debatable.

      Delete
    21. The claim does not require sufficiency for a 1 second erasability - it claims erasability within 5 minutes.

      "An airplane that can fly at a speed upto 1000 km/hour" does not require that the airplane can also fly at 1 km/hour... It just imposes an upper limit.
      Nor does "An airplane that can fly at a speed over 1000 km/hour" require that it can fly at 50000 km/hour.

      Delete
    22. Thank you so much for your effort, Roel!

      However, I do not think that this is a good analogon. The ability to fly is an intrinsic property of an airplane. It is absolutely clear that flying requires a certain minimum speed.

      Erasability is definitely not an intrinsic property of an ink, though. Erasability is a very special property, like the ability of a stealth aircraft to avoid detection by a radar.

      If an airplane was claimed, with the ability to avoid detection by radar when travelling with a speed up to 1000km/hour, and if it was absolutely clear that avoiding detection is impossible at speeds less than, e.g., 800km/hour, I would definitely see an A83-problem.

      I think that there is no doubt that the claim of the exam covers embodiments that are not sufficiently disclosed. Maybe both positions can be argued, but to expect to process these thoughts with only 3.5min/questions (reading excluded!) is unrealistic. I think this question should be neutralized.

      Delete
  11. Thanks so much for your effort! I’m kind of frustrated about the claim analysis part, especially the second claim analysis part. Luckily, comparing my answers to yours, I’ve passed the exam. However, in my point of view, previous exams were way easier.

    ReplyDelete
  12. We have added the Description of the application and Document D1 and D2 (none of them had any Figures) as an Annex to the end of the blog post.

    ReplyDelete
  13. Thank you for your Analysis.

    About Q20.2:
    In retrospect, while I also think that the EPO's point was that Art. 84 is not a ground for opposition, I came across decision T862/11 during the exam. There it says (translated from German):

    Insofar as inventions are defined by claims which contain unclear features, the invention can possibly only be understood by the result to be achieved. This effect, is then considered under Article 83 in order to assess the reproducibility of the invention.

    Further details on this can also be found here: https://www.epo.org/law-practice/legal-texts/html/caselaw/2022/e/clr_ii_c_3_2.htm

    So could one not also argue that Q20.2 is true?

    ReplyDelete
    Replies
    1. 20.2 is not about Art. 84, but about Art. 83. Case Law is very strict in taking care that an alleged Art. 83 attacks is not an Art. 84 attack in disguise.

      See Guidelines F-III, 1:

      "1. Sufficiency of disclosure and clarity
      An ambiguity in the claims may lead to an insufficiency objection. However, ambiguity also relates to the scope of the claims, i.e. Art. 84 (see F‑IV, 4). Normally, therefore, an ambiguity in a claim will lead to an objection under Art. 83 ONLY IF THE WHOLE SCOPE of the claim is affected, in the sense that it is impossible to carry out at all the invention defined therein. Otherwise an objection under Art. 84 is appropriate (see T 608/07, T 1811/13)."

      As there is a clear example in the application, there may be an Art. 84 problem (too broad, not supported over whole scope), but there is no Art. 83 problem.

      Delete
  14. If you are interested whether an AI would have passed this Pre-Exam: we have asked ChatGPT to give its answers: see:
    http://pre-exam.blogspot.com/2023/03/chatgtp-4-finds-pre-exam-legal-part.html
    and
    http://pre-exam.blogspot.com/2023/03/chatgtp-4s-attempt-at-parts-3-4-of-pre.html

    Sander, Nico, Roel

    ReplyDelete
  15. A general question:

    To me, especially the fourth part appears extremely ambiguous and extremely vague in a great number of points. Does the EPO take notice of the concerns raised in this forum? Is there any way to draw their attention to the problems discussed here?

    It is just incredibly frustrating to study hard for months and then to get questions that are so unclear, that even after days of discussing no clear answers can be found.

    ReplyDelete
    Replies
    1. I cite from the Tutor report on the EQE 2022 in epi Information 4/2022 (page 38, right column):

      "At the Tutor Meeting, it was indicated that all information
      that was brought to the attention of the Committee and
      the Examination Board, as well as posts and comments on
      blogs, were carefully considered to see whether any neutralization were required."

      which was also repeated by the Pre-Exam committee (page 40, left column):

      "Stefan Götsch [Pre-Exam Committee) indicated that the Committee and Examination
      Board consider statistics, blog posts and comments when
      deciding on a possible neutralization of a statement or question, as these may indicate possible problems and ambiguities. The information is also used in the process of the Committee"

      Also the Tutor report for earlier EQEs mention the sane, e.g., the Tutor Report on EQE 2021 in epi Information 4/2021 provides on page 28, right column:

      "The Examination Board tried to be as fair as possible in
      the marking to take into account the circumstances. E.g.,
      for paper B there was not just one answer to get close to
      full marks. Also, apart from results from the pre-marking,
      comments on the blogs and other information available
      to the Examination Board were used in finalizing the marking scheme. [...]"

      Delete
    2. (you can find the archived issues of epi Information on
      https://patentepi.org/en/epi-information/archived-issues.html)

      Delete
    3. It's great that EPO is open to listen to the voice through this blog as well! Since if it wasn't then unnecessarily complicated appeal proceedings would be increased.

      Delete
  16. In my opinion it was quite hard for me as a non-chemist to first get and subsequently keep the functions of the several substances in mind. This led to a time problem which did not happen at all while studying. I guess this was a huge advantage for chemists!

    ReplyDelete
  17. Isn't it strange to put a "preferably" at the end of one of the features of an enumerated list, and then have it refer to the next item? Should it not have been at the beginning of the next item?

    II.1 was given as:
    II.1. An erasable ink composition comprising:
    (a) from about 40% to about 90% w/w of water;
    (b) from about 1.5% to about 30% w/w of a colouring agent;
    (c) from about 0.3% to about 30% w/w of a releasing agent; and preferably
    (d) from about 1% to about 15% w/w of a film-forming polymeric binder.

    This does not test a candidate's understanding of "preferably". It just tests close reading of an unusually formatted claim.

    Why was it not:
    II.1 was given as:
    II.1. An erasable ink composition comprising:
    (a) from about 40% to about 90% w/w of water;
    (b) from about 1.5% to about 30% w/w of a colouring agent;
    (c) from about 0.3% to about 30% w/w of a releasing agent; and
    (d) preferably, from about 1% to about 15% w/w of a film-forming polymeric binder.

    ReplyDelete
    Replies
    1. What is the effect of the "about"s in all these claims?

      Guidelines say in F.IV.4.7.1 Where terms such as "about" or "approximately" are applied to a particular value (e.g. "about 200°C" or "approximately 200°C") or to a range (e.g. "about x to approximately y"), the value or range is interpreted as being as accurate as the method used to measure it. If no error margins are specified in the application, the same principles described in G‑VI, 8.1 apply, i.e. the expression "about 200°C" is interpreted as having the same round-off as "200°C". If error margins are specified in the application, they must be used in the claims in place of the expression containing "about" or similar terms.

      I do not see where and how I had to use that?

      Delete
    2. I don't know. I do not think the paper would have been that different if all the "about"s would have been absent (only really relevant for 17.3) - but I must admit that I did not check it very carefully: any other opinions.

      Note that the general principle is that, without any indication to the contrary, the accuracy with and without "about" is the same - it just indicates tolerances, as given by the technical accuracy and by "the general convention in the scientific and technical literature is applied: the last decimal place of a numerical value indicates its degree of accuracy" (G: G-VI, 8.1).

      The references in F-IV, 4.7.1 cited above by Anonymous seems to relate to T 1986/14 cited in the Case Law Book II.E.1.5.2, which indicates that only the end points as given in the original accuracy are disclosed, despite the use of "about":

      "2.1 Claim 6 of the main request was amended by including the features "glycerin in an amount ranging from 50% to 90% by weight of the composition" and "sorbitol in at least 0.1% by weight of the composition".

      2.2 Glycerin in an amount ranging from 50% to 90% by weight of the composition

      The appellant argued that the amount of glycerin in claim 6 found a basis in the passage on page 9, lines 17-19, of the application as originally filed, which read "glycerin moisturizer can be present individually in an amount ranging from about 50.00% to about 90.00% by weight".

      It was undisputed that 50% and 50.00% differ in their accuracy. For this reason, 50.00%/90.00%, on their own, cannot not provide a basis for the features 50% or 90%.

      The applicant argued, however, that the use of the term "about" in the passage mentioned above indicated that it was not intended to restrict the claimed amount to ranges defined by end-points with four significant figures.

      The feature "about 50.00% to about 90.00%" discloses a range with two end-points, namely 50.00% and 90.00%, and an area of undefined boundaries around them. No other end-point, such as 50% or 50.0%, is either implicitly or explicitly disclosed. For this reason, the passage cited cannot provide a basis for the afore-mentioned feature."

      Delete
  18. My summary from the discussion on essential features in 16.1 and 16.4:

    16.1 The following feature is described as essential in the description: The modified siloxane-containing releasing agent comprises less than 1% w/w of colloidal silica.

    One can answer T because it is essential in the specific embodiment wherein the releasing agent includes a modified siloxane, i.e., to achieve that “”the erasing is even more improved” [006], for which “Ii is a finding of the inventors” that “the releasing agent includes a modified siloxane” provides that.

    One can answer F because it is only optional (for the invention in the broadest sense , i.e. to “allow the erasable ink composition to be erased from a surface made of paper” [006], first sentence; and with reference to claim I.1/I.2), so not essential, not that the releasing agent includes a modified siloxane

    &

    16.4 The following feature is described as essential in the description: The releasing agent is a modified siloxane.

    One can answer F because it is only optional (for the invention in the broadest sense, i.e. to “allow the erasable ink composition to be erased from a surface made of paper” [006], first sentence; and with reference to claim I.1/I.2), so not essential, not that the releasing agent includes a modified siloxane

    One can answer T because it is essential in the specific embodiment wherein the erasing is even more improved – if that is understood as the the technical problem with which the application is concerned


    Note that 16.1 and 16.4 are dependent: if one considers that “The releasing agent is a modified siloxane” and concludes that 16,4 is T, then also “The modified siloxane-containing releasing agent comprises less than 1% w/w of colloidal silica” is essential and 16.1 is also T.


    So it seems that one cannot unambiguously answer T or F for 16.1 nor 16.4.

    The issue seems to be that neither 16.1 nor 16.4 refer to a specific claim, while essential features are only defined with reference to a specific claim or at least with reference to “the invention” which is usually interpreted as “the invention as presented in the description solving the (subjective) technical problem with which the invention is concerned”, i.e. “the invention as it should have been claimed in the independent claim”:

    Guidelines F-IV, 4.5.2: ”a CLAIM must be comprehensible from a technical point of view, [and] it must define clearly all the essential features OF THE INVENTION”

    Guidelines F-IV, 4.5.2: “Essential features OF A CLAIM are those necessary for achieving a technical effect underlying the solution of the technical problem with which the application is concerned (the problem usually being derived from the description). The independent claim(s) must therefore contain all features explicitly described in the description as being necessary to carry out the invention. Any features which, even if consistently mentioned in the context of the invention throughout the application, do not actually contribute to the solution of the problem are not essential features.”

    Guidelines F-IV, 4.5.3: “It is not necessary to include all details of the invention in the independent claim. Thus a certain degree of generalisation of the claimed features may be permitted, provided that the claimed generalised features as a whole allow the problem to be solved. In this case a more specific definition of the features is not required”.

    ReplyDelete
  19. My summary on water and water-based solvent system (D2, [002]) may be illustrated with two T/F statements.

    The issue is that it is nowhere explicitly defined what the composition is of a "the water-based solvent system": is it 100% water or not?

    For each of the statements, indicate whether the statement is true (T) or false (F):
    1. A water-based ink is composed of 100% water.
    2. A water-based solvent system is composed of 100% water.

    As 1.is F (as the description provides water-based inks that comprise more than must water) that can be considered to provide a definition of water-based, then logic provides that there is no reason to assume that 2. is true (also in view of the absence of a definition of ‘water-based solvent system’ ).

    A contrario reasoning: If 2 is T, then logic provides that also 1 is T – but as 1 is not T, 2 cannot be T.

    D2, [002] reads: “The simplest erasable ink composition for use in a writing instrument, such as a pen, comprises a solvent and a pigment. Preferably, said pigment has a flake-like morphology. The pigment, whatever its colour, is dispersed in the solvent. It seems that for the most efficient systems, the erasable ink compositions include from about 5% to about 20% w/w of the flake-like pigment and from about 74% to about 94% w/w of the water-based solvent system. Such erasable ink compositions exhibit an erasability of at least 80% and from about 74% to about 94% w/w OF THE WATER-BASED SOLVENT SYSTEM. ”

    In view of the T/F arguments above, can NOT be understood to directly and umambigously disclose:
    “…and from about 74% to about 94% w/w OF WATER“ nor
    “…and from about 74% to about 94% w/w OF THE SOLVENT, THE SOLVENT BEING WATER” nor
    “…and from about 74% to about 94% w/w OF THE SOLVENT- BASED SOLVENT SYSTEM, THE BASED SOLVENT SYSTEM SOLVENT BEING 100% WATER”.

    So, the % w/w of water present in the water-based solvent system in D2 is not disclosed, so that the it cannot destroy novelty of the specific ranges of % w/w of water in the claims.

    ReplyDelete
    Replies
    1. Thanks. My two cents: Is not 'dispersed' incorrect for a 'solvent'-system? Should it not have read 'dissolved'. Does not dispersing lead to a suspension, rather than a solution?

      Delete
    2. pigments can't be dissolved (not in water and not in solvent).
      It will be a water-based pigment dispersion, which is the basis for any water-based pigment ink.

      Delete
    3. I see. But how can I know?

      Delete
    4. Probably by being an expert in chemistry/inks.

      But I don't know if this information is important to answer the questions?

      Delete
  20. I found both claim analyses parts significantly more difficult than previous year (to an extent, I found the legal part trickier but mostly in relation to the 2022 pre-exam which evidently much easier as seen by the high pass rate).

    Disclaimer: Due to anxiety I have not read any above comments nor the proposed solutions and I have not backtracked any questions. I am commenting completely based of my own perspective and only based on my memory. Apologies for any inaccuracies in advance. You could say I am providing my feelings as a "well prepared candidate". When completing old pre-exams I had scores ranging from 82-94 points. Admittedly this was under no pressure but I do think at best my score on this exam will be around 70-80 points. Consequently, I thought the entire claim analyses was much more difficult than previous papers..

    In general terms part 4 seemed too chemistry-specific. Many materials/compositions and different names and effects... Understanding the invention and the prior art was hard.

    I have two specific examples that I found difficult in particular:
    - The question on essential features "The following feature is described as essential in the description: The modified siloxane-containing releasing agent comprises less than 1% w/w of colloidal silica." I have never seen something similar and I could not find a suitable example in the guidelines under time pressure. The description stated that 1% w/w of colloidal sillica was clearly necessary when using a modified siloxane-containing releasing agent, but the modified siloxane-containing releasing agent was in itself not described as essential. I would have loved to be nuanced during the exam but T/F can never let me demonstrate I understood this.
    - The "sufficiency of disclosure attack" question in opposition was very surprising and on a topic I had never seen before. I cannot remember any course material or previous questions on this throughout my studies. GL F-III, 1 did not shed light on the matter in such a way that I could easily lean towards either answer with statements like: "regard must be had to the facts and evidence of the particular case".

    Although I only provided two examples I will stress that my feeling of being able to understand the invention and the prior art was much reduced by the technical field.

    ReplyDelete
  21. Did all of you fill out the feedback survey? Does this make any sense?

    ReplyDelete
    Replies
    1. Yes, I did. You can rate the difficulty of both the legal part and the claim part. At least it is some sort of feedback we can give the EPO.

      Delete
  22. As mentioned in the previous comments, I found the question concerning "essential features" to be difficult since I did not know how to interpret the statements. The description stated that 1% w/w of colloidal silica was clearly necessary WHEN using a modified siloxane-containing releasing agent, but the modified siloxane-containing releasing agent was in itself described as optional. Thus, my reasoning was that IF a modified siloxane-containing releasing agent was used, then indeed it would be essential that it comprises less than 1% w/w of colloidal silica. But, as a whole, "The modified siloxane-containing releasing agent comprises less than 1% w/w of colloidal silica" was for me NOT essential, as the modified siloxane-containing releasing agent was considered optional.

    In my opinion, part 4 of the pre-examination was quite difficult to understand partly due to the (in my view) unclear definitions of the different substances. Obviously, based on my comment, I did not study chemistry, and thus got a bit confused with all the substances, ranges etc.

    ReplyDelete
  23. My feeling is that part 4, which is so chemistry-specific, put all the non-chemists at a huge disadvantage. Not just it's a completely different file that I work in, but also since so many chemical components were not the same and it was difficult to compare the documents, some knowledge of chemistry was a prerequisite, however, we were always told that specific knowledge should not be used for answering the exams. Further, as some details seems to be missing, as Roel mentioned previously.

    It is also not clear why a main exam level for Paper C does not test the same questions as we were tested at pre-exam, which is supposed to be at a "basic" level.

    ReplyDelete
  24. Hi, thanks for providing your answers! apologies if the below was discussed in other comments, I could not read them all.

    I do not agree with the answer to Q 18.3:
    Claim I.3 is novel over D2.

    I think the answer should be false.

    I.3 when written out reads:
    An erasable ink composition comprising:
    (a) water;
    (b) a colouring agent;
    (c) a releasing agent; and
    (d) a film-forming polymeric binder,
    wherein the water is present in an amount ranging from about 40% to about 90% w/w.

    D2 discloses a composition comprising a solvent and a pigment the pigment (first sentence of [002]). the amount of pigment is 5-20% w/w and the amount of water-based solvent system is 74-94% w/w based on weight of the composition (4th sentence of [002]). this implies that the solvent of the first sentence is a water-based solvent system. water-based solvent system means a solvent which comprises water and other components. the pigment is not part of the water based solvent system.

    the answer above argues: "D2 described “about 74% to about 94% w/w of the water-based solvent system”, but it does not indicate which percentage of that is water and what percentage is pigment." however, this is not true because the pigment is not part of the water-based solvent system. this is clear from the 4th sentence.

    I agree that the water-based solvent system does not specify what else is included other than water. however, one would know that at least water is there and one viable option is the water based solvent system is 100% water. I think that 100% water in the water-based solvent system is at least inherently disclosed. therefore, D2 has disclosed, (a) water in an amount of 74-84% w/w based on weight of the composition and (b) a colouring agent.

    D2 has also disclosed a dispersant which is a polymer (first two sentences of [003]) and acts as film forming and releasing agent (last sentence of [003]). hence, D2 has disclosed (c) a releasing agent; and (d) a film-forming polymeric binder.

    polymeric binder and polymeric dispersant are essentially synonyms. I agree that a person with no chemistry background would have a hard time recognising that. however, (1) the fact that polymeric binder and polymeric dispersant can be the same thing is clarified by the last sentence of paragraph [003] of D2; and (2) if a feature has a different name than in the prior art, but does the exact same thing, the different name is usually not enough for novelty.

    it happened in previous Pre-exams that mechanical terms where considered synonyms even though it was not 100% explicit they were the same but only suggested in the text. for example in the second part of the Pre-EQE of 2021 (tablet protection device), the terms fold and hinge where considered equal even if it was not explicitly mentioned. however, they served the same function.

    I do believe this question would have been hard without a chemical background. however, I do also believe that 74 -94% w/w of a water-based solvent system would be usually interpreted to inherently disclose 74-94% w/w of water. furthermore, in previous exams, terms where considered synonyms when features served the same function.

    if the answer was supposed to be T, then it is very frustrating that the answer would be against (1) common practice of interpreting a chemical claim and (2) interpretations of previous exam.

    I genuinely hope this statement (and similar ones) will be neutralised.

    thanks again for the animated discussion.

    ReplyDelete
    Replies
    1. I am wondering what the chances for an appeal would be in case all these unclear statements are not neutralized?! Especially wrt questions 16, 18 and 20.

      Does anybody have any experience/insights regarding appeals against epo exams?

      Delete
  25. another little rant from me: I found the statements of claim 20 weirdly formulated. for example "In an opposition procedure, an opponent will likely be successful with an attack under Article 83 EPC against claim II.3." will likely be successful is not a very clear phrasing. the statement does not specify what the attack is.

    I understand that they are essentially asking "does the claim comply with A83 EPC?". however, this is not exactly the same. in opposition the burden of proof for an attack under A83 is with the opponent. the opponent has to raise serious doubts supported by verifiable facts. there is a lot of case law in which the OD or BoA rejected attacks under A83 because the opponent did not provide additional data.

    on the other hand, in examination the burden of proof is with the applicant to show that the invention is sufficiently disclosed.

    I know this does not necessarily matter in answering the question, but I think that the phrasing of the statement could have let to much more overthinking than necessarily.

    if the Pre-EQE committee wanted to test candidate's knowledge of compliance of a claim with A83, they should have just asked that. I find the phrasing of the statement unnecessarily unclear. this was the last question after a full day of exams and was not easy to answer. a clearer statement would have been more fair.

    ReplyDelete
  26. Anyone else sees a change in the buttons in myEQE? Some people saw that enrolment to the main examination is now possible in their myEQE-account.

    I see that the button to the pre-exam is activated for me, although I did so well that I would be very surprised if I failed...... I am now very worried!

    ReplyDelete
    Replies
    1. In principle candidates may enrol as from 3 April 2023 (see https://www.epo.org/learning/eqe/enrolment.html), but you can only enrol in MyEQE after you have passed the Pre-Exam.

      If you saw that enrolment was enabled, my bed would be that that is an automatic enabling due this date, 3 April 2023.

      For the subsequent deactivation, I would consider it more likely that the results are not yet ready, e.g., they may still or again be subject to review or to correction after decisions have been taken as to which statements are neutralized - which would also explain why the enrolment for Main Exam 2-24 is (again) disabled for all candidates that sat Pre-Exam 2023.

      Let's hope that the results come out soon and that statements that have been serious candidates for neutralization will show to be neutralized!

      Delete
    2. We all hope so... Thanks again for your effort, Roel!

      Delete
  27. It's deactivate for me also.

    ReplyDelete
  28. Same for me. I can enroll for the pre-Exam although according to Delta Patents I should have passed the exam by at least 8-12 points…

    ReplyDelete
  29. Friend of mine just said his EQE button was activate yesterday and deactivated today again, so it seems it's no indication of pass/fail.

    ReplyDelete
  30. Was the same in my case: Button was activated yesterday, now deactivated again. Appears to be no indication.

    ReplyDelete
  31. For me it's still activated (the pre-EQE button), I wonder what's going on!

    ReplyDelete
  32. I wonder why they would activate the pre-exam button for a candidate that actually passed the exam. Maybe it is an indication?

    ReplyDelete
  33. I don't know if my main exam enrollment button was activated on April, 3 since I did not connect then, but the main exam enrollment button is now deactivated and the preexamination enrollment button is now activated. Is that the case for everybody?

    ReplyDelete
  34. That's the same for me, I never seen the main exam button activated, but Roel mentioned that it's probably irrelevant.

    ReplyDelete
  35. Same for me.

    I suppose the portal looks for a value indicating that participants passed the PreEQE, doesn't find one yet and thus opens the enrolment for PreEQE?

    ReplyDelete
  36. Same here. What a treat, first very unclear statements, second very long waiting time, third driving everyone nuts with the activated buttons :-DDDD

    ReplyDelete
  37. This is taking too long......

    ReplyDelete
  38. I am also wondering... Any ideas, what they need four weeks for? Roel?

    Even if they decide to neutralize certain statements, shouldnt this be done in one or two days?

    ReplyDelete
  39. I just noticed that the button to the pre-exam is deactivated, so does that mean that I passed?

    ReplyDelete
  40. Also my preex-button is deactivated, but main exam is not yet activated. Is yours activated?

    ReplyDelete
  41. Same here, both buttons are deactivated now.

    ReplyDelete
  42. No results yet. Delay due to EQE getting EPAC results out?

    ReplyDelete
  43. Can we get them today please:-(?

    ReplyDelete
  44. Has it ever taken longer than this year? So annoying...

    ReplyDelete
  45. The results are available !!

    ReplyDelete
    Replies
    1. I'm not seeing any results on myEQE. I hope that doesn't mean I have failed!

      Delete
    2. I'm not also seeing any results..

      Delete
    3. Are you for real or just trolling us?

      Delete
    4. get a life..

      Delete
    5. Definitely not true until now.

      Delete
  46. Has anyone else received their results? If so, are they good or bad? I'm just wondering why I have not yet received mine.

    ReplyDelete
  47. I dont see my score, but my main exam button is now activated so i supposed it was ok

    ReplyDelete
  48. I don't have a score and neither my pre-exam nor my main exam button are activated. Very worried!

    ReplyDelete
  49. I don't see any results. Are some people pulling jokes on us that they have received theirs? Funny 😂

    ReplyDelete
  50. Seriously, is this a joke?

    ReplyDelete
  51. I guess we'll have another weekend without results

    ReplyDelete
  52. I think so too... The EPO should consider investing time and resources into setting up a proper exam in the first place, not into fixing it afterwards. Or maybe they just dont care under how much stress most candidates are because of this exam.

    ReplyDelete
  53. I can't really operate properly, checking my phone 100 times an hour. Come on

    ReplyDelete
    Replies
    1. That's too much 😅

      Delete
    2. Too much is subjective, so you will get a clarity objection Art. 84

      Delete
  54. This is taking too long .. but it may be a good sign they must be discussing neutralizations

    ReplyDelete
  55. In 2021 or 2019 it took more than a month

    ReplyDelete
  56. No.

    2021: exam date: 1/3/2021, results out: 31/3/2021.

    2019: exam date: 25/2/2019, results out: 12/3/2019.

    ReplyDelete
    Replies
    1. Indeed, I was gonna say it!
      If we don't get it today then this will be the longest waiting time in a while

      Delete
    2. 2022: exam date 18/3/2022, results out: 8/4/2022
      (Complete part 3 + 1 question in part 4 = 30 marks neutralized)

      Delete
    3. well in 2021 it was more than 4 weeks so.. maybe next thursday? or wednesday

      Delete
  57. If for them it is taking this long (4 weeks) to decide on the answers to the questions we have answered in four hours, they should neutralise the whole paper.

    ReplyDelete
    Replies
    1. Why? Candidates should be smart enough to know the answers even though the examiners themselves don't 😂

      Delete
  58. Please use your name, or a nickname, if you post comments. It is difficult to distinguish all these Anonymous-es... ;)

    ReplyDelete
  59. Dear Roel, thanks again for your answers. One question: Isnt there a Delta Patents course for the main exam (part D distance learning) that starts in May? Do people already enroll for this course, even though the results are not yet available?

    ReplyDelete
    Replies
    1. Yes, we have. See https://www.deltapatents.com/cursus/d-distance-learning/

      If you enrol to the course and it would unexpectedly happen that you did not pass the Pre-Exam, we can move your enrolment to the next year.

      Delete
  60. I didnt come here since the article was posted... I didn't realise the amount of stress people go through

    ReplyDelete
  61. Will they release the results in the coming 2 hours of the week? 😶
    Pleaaaaasssseeeeeeee I don't want to have another stressful weekend

    ReplyDelete
    Replies
    1. It’s 4:30 I don’t think they will release the results now …

      Delete
  62. Welcome to another wonderful weekend of endless wait! Thanks EPO ❤️

    ReplyDelete
  63. This is just too much......

    ReplyDelete
    Replies
    1. Responding within one minute shows it all

      Delete
    2. Responding within five weeks too

      Delete
  64. One more day gone

    ReplyDelete
  65. Let’s see the good part.. this must be the last week of wait

    ReplyDelete
    Replies
    1. Why?? Maybe it will take another month.

      Delete
    2. Stop being agressive I was trying to cheer up people you are starting to act like crazy for this exam

      Delete
    3. you're totally right, we should all calm down. but honestly, if there's an exam to act crazy about, then it's this one.

      Delete
    4. Can't calm down knowing they are spending 5 weeks thinking of the answers of T/F questions which we solved in 4 hours. This is insane on its own.

      Delete
    5. The thing is that technically, they had to already fix the answer before they release the questions to the exam and technically we could get the result as soon as we submit it. However, the only thing they have to do is discussion for the questions to be neutralised. But seriously this would take this much long? I think they have to release it before the term for the enrolment of pre-exam starts.

      Delete
    6. I don’t understand what it is that takes so long. In 2021 it took more than four weeks and there was not a single neutralization.

      Delete
    7. Acting crazy won't change the day of release of results nor your results, but keep doing it if you feel better

      Delete
    8. The pass rate must have been so low, I think they are harmoinizing the marks

      Delete
    9. Not sure. The EPAC pass rate was 35% and they didn't care to change some things to raise it.

      Delete
    10. the EPAC does not lead to another exam.. a very low pass rate for the pre-Exam leads to few candidates to the main examination..

      Delete
    11. @Anonymous 18 April 2023 at 21:31

      That is not correct.

      In 2021, complete Q.10 was neutralized. Please refer to the Examiner's Report or our P-book:
      "Question 10 mistakenly refers to applicant B and applicant A while it was intended to refer to only one applicant B different from applicant A of question 4. To avoid any disadvantage due to this mistake, it has been decided that for this question 10 all answers are awarded the full marks of 5 points."

      Several others were suggested to be neutralized in comments on our blogs. The Examination Board will have considers all those comments (see Tutor's reports in epi Information), even though they did not lead to further neutralizations in the original marking.

      And, not neutralized in the original marking, but successful in appeal and neutralized for those that appealed - also see our P-book:
      Statement 12.4 "Claim I-8 is unclear due to the use of the term low": D 2/21, D 4/21, D 7/21 (EN), D 6/21 (DE), D 5/21, D 3/21 (FR). The Disciplinary Board considered that the statement had to be neutralized.

      Also, a successful appeal was filed in respect of statement 19.4: D 5/21 (FR) - Pre-Exam 2021, statement 19.4 "Conformément à l'article 123(2) CBE, il y a une base pour modifier comme suit la revendication IV.1 de la demande déposée initialement: [...]" / "Under Article 123(2) EPC, there is basis for amending claim IV.1 of the originally filed application as follows: [...]" The Disciplinary Board considered that the statement had to be neutralized.

      Delete
    12. My guess would be that they have decided on what to neutralize but that examining complaints and possible new things brought to their attention caused a delay (it is later than earlier years). Maybe also the randomization of the sequence of questions (legal part) and statements within a question (legal and claims parts causes complications when executing the neutralization or double-checking the results it it. Let's hope the results now come soon! (But I will no longer refresh the webpage where the pre-exam 2023 examiner's report will appear a few times per day, but check it only once per day. I assume that candidates get an email immediately after the results are available in myEQE, so just checking the email that you used for Wiseflow would be enough).

      Delete
  66. Let's welcome another weekend without results, yay!

    ReplyDelete
  67. The enrolment button to the main EQE is open for me. (But no result yet.)

    ReplyDelete
    Replies
    1. Same for me. Thanks for sharing. I stopped checking regularly.

      Delete

Post a Comment

1 – 200 of 259 comments Newer Newest